LSAT and Law School Admissions Forum

Get expert LSAT preparation and law school admissions advice from PowerScore Test Preparation.

User avatar
 Dave Killoran
PowerScore Staff
  • PowerScore Staff
  • Posts: 5853
  • Joined: Mar 25, 2011
|
#27318
Complete Question Explanation

The correct answer choice is (A)

Answer choice (A) is the correct answer choice. As shown in the 2-5 Dist Possibility in the setup discussion, F and S can be the only two flavorings in the appetizer recipe.

Answer choice (B) is incorrect because from the third rule either S or T must be included in the appetizer recipe.

Answer choice (C) is incorrect because F and N cannot be included the same recipe per the second rule.

Answer choice (D) is incorrect because from the last rule N and G must be included in the same recipe.

Answer choice (E) is incorrect because from the first rule the appetizer recipe includes at most three flavorings.

Get the most out of your LSAT Prep Plus subscription.

Analyze and track your performance with our Testing and Analytics Package.